当前位置: 华文星空 > 知识

如何计算这个极限?

2021-03-06知识

\lim_{n \rightarrow \infty}{\frac{\ln^2{n}}{n}\sum_{k=2}^{n-2}{\frac{1}{\ln k\ln(n-k)}}}

@23Square 给出的答案已然完备。

其中一个方向的放缩比较简单:

{\frac{\ln^2{n}}{n}\sum_{k=2}^{n-2}{\frac{1}{\ln k\ln(n-k)}}}\geq1-\frac{3}{n}

在这里给出另一个方向的放缩的一种思路:

由Cauchy-Schwarz不等式

{\frac{\ln^2{n}}{n}\sum_{k=2}^{n-2}{\frac{1}{\ln k\ln(n-k)}}}\leq{\frac{\ln^2{n}}{n}\sum_{k=2}^{n-2}{\frac{1}{\ln^2{k}}}}

再由O’Stolz定理及迫敛性准则,易得

\lim_{n \to \infty} {\frac{\ln^2{n}}{n}\sum_{k=2}^{n-2}{\frac{1}{\ln^2{k}}}}=1

由迫敛性准则,所求极限为1.